LSAT and Law School Admissions Forum

Get expert LSAT preparation and law school admissions advice from PowerScore Test Preparation.

User avatar
 Dave Killoran
PowerScore Staff
  • PowerScore Staff
  • Posts: 5853
  • Joined: Mar 25, 2011
|
#87992
Complete Question Explanation
(The complete setup for this game can be found here: lsat/viewtopic.php?f=162&t=5831)

The correct answer choice is (B).

The condition in the question stem establishes that F is visited by S, which from the second rule is the only executive that visits F:

G4-Q23-d1.png

This block must be either first or second. From the third rule, Q must visit a site before R and T. Thus, Q cannot visit the third site, and must also be either first or second. Because R and T cannot visit the same site as Q, and they cannot visit the same site as S (because S is the only executive visiting F), R and T cannot be either first or second, and they have no choice but to visit the third site. Accordingly, answer choice (B) is correct.
You do not have the required permissions to view the files attached to this post.

Get the most out of your LSAT Prep Plus subscription.

Analyze and track your performance with our Testing and Analytics Package.